Difference between revisions of "2018 AMC 12B Problems/Problem 17"
m (→Solution 1) |
|||
Line 39: | Line 39: | ||
Remark: This also gives an intuitive geometric proof of the mediant. | Remark: This also gives an intuitive geometric proof of the mediant. | ||
+ | |||
+ | ==Solution 5 (Using answer choices to prove mediant)== | ||
+ | As the other solutions do, the mediant <math>=\frac{9}{16}</math> is between the two fractions, with a difference of <math>\boxed{(A) 7}. Suppose that the answer was not </math>A<math>, then the answer must be </math>B<math> or </math>C<math> as otherwise </math>p<math> would be negative. Then, the possible fractions with lower denominator would be </math>\frac{k-11}{k}<math> for </math>k=12,13,14,15<math> and </math>\frac{k-13}{k}<math> for </math>k=14,15,<math> which are clearly not anywhere close to </math>\frac{4}{7}\approx 0.6$ | ||
==See Also== | ==See Also== |
Revision as of 00:58, 19 February 2018
Contents
Problem
Let and be positive integers such that and is as small as possible. What is ?
Solution 1
We claim that, between any two fractions and , if , the fraction with smallest denominator between them is . To prove this, we see that
which reduces to . We can easily find that , giving an answer of .
Solution 2 (requires justification)
Assume that the difference results in a fraction of the form . Then,
Also assume that the difference results in a fraction of the form . Then,
Solving the system of equations yields and . Therefore, the answer is
Solution 3
Cross-multiply the inequality to get
Then,
Since , are integers, is an integer. To minimize , start from , which gives . This limits to be greater than , so test values of starting from . However, to do not give integer values of .
Once , it is possible for to be equal to , so could also be equal to The next value, , is not a solution, but gives . Thus, the smallest possible value of is , and the answer is .
Solution 4
Graph the regions and . Note that the lattice point (9,16) is the smallest magnitude one which appears within the region bounded by the two graphs. Thus, our fraction is and the answer is .
Remark: This also gives an intuitive geometric proof of the mediant.
Solution 5 (Using answer choices to prove mediant)
As the other solutions do, the mediant is between the two fractions, with a difference of ABCp\frac{k-11}{k}k=12,13,14,15\frac{k-13}{k}k=14,15,\frac{4}{7}\approx 0.6$
See Also
2018 AMC 12B (Problems • Answer Key • Resources) | |
Preceded by Problem 16 |
Followed by Problem 18 |
1 • 2 • 3 • 4 • 5 • 6 • 7 • 8 • 9 • 10 • 11 • 12 • 13 • 14 • 15 • 16 • 17 • 18 • 19 • 20 • 21 • 22 • 23 • 24 • 25 | |
All AMC 12 Problems and Solutions |
The problems on this page are copyrighted by the Mathematical Association of America's American Mathematics Competitions.